Mo Logo [Home] [Lexikon] [Aufgaben] [Tests] [Kurse] [Begleitmaterial] [Hinweise] [Mitwirkende] [Publikationen]

Mathematik-Online-Aufgabensammlung: Lösung zu

Aufgabe 858: Abschätzungen zweier Summen


A B C D E F G H I J K L M N O P Q R S T U V W X Y Z

Seien $ \mbox{$x_1,\dots,x_n\in\mathbb{R}_{>0}$}$. Zeige die folgenden Ungleichungen.

(i)
$ \mbox{$\left(\sum_{i = 1}^n x_i\right)^2 \;\leq\; n\cdot\sum_{i = 1}^n x_i^2\; .$}$
(ii)
$ \mbox{$\left(\sum_{i = 1}^n ix_i\right)^{3/2} \;\leq\; \frac{n(n+1)}{2}\cdot\sum_{i = 1}^n x_i^{3/2}\; .$}$

(i)
Mit $ \mbox{$y_i = 1$}$ für $ \mbox{$1\leq i\leq n$}$ wird mit Cauchy-Schwarz
$ \mbox{$\displaystyle
\begin{array}{rcl}
\left(\sum_{i = 1}^n x_i\cdot 1\right...
...ight)\vspace*{2mm} \\
& = & n\cdot\sum_{i = 1}^n x_i^2\; . \\
\end{array}$}$

(ii)
Mit $ \mbox{$y_i = i$}$ für $ \mbox{$1\leq i\leq n$}$, mit $ \mbox{$p = 3$}$ und $ \mbox{$q = 3/2$}$ wird mit Hölder
$ \mbox{$\displaystyle
\sum_{i = 1}^n i x_i \;\leq\; \left(\sum_{i = 1}^n i^3\right)^{1/3}\left(\sum_{i = 1}^n x_i^{3/2}\right)^{2/3}\; ,
$}$
woraus wir
$ \mbox{$\displaystyle
\begin{array}{rcl}
\left(\sum_{i = 1}^n i x_i\right)^{3/...
...& \frac{n(n+1)}{2}\cdot\left(\sum_{i = 1}^n x_i^{3/2}\right) \\
\end{array}$}$
ersehen.
(Autoren: Künzer/Martin/Nebe)

[Zurück zur Aufgabe]

  automatisch erstellt am 7.  6. 2005